Intermediate Value Theorem Converse

  • Thread starter Thread starter KF33
  • Start date Start date
  • Tags Tags
    Theorem Value
Click For Summary
The discussion centers on the validity of the Converse of the Intermediate Value Theorem (IVT). The converse states that if there exists a number c in [a,b] such that f(c)=k for any k between f(a) and f(b), then the function f must be continuous on [a,b]. Participants express skepticism about the truth of this converse, suggesting it may be false due to the potential lack of continuity in the function. The conclusion drawn is that the converse does not necessarily hold true without the continuity condition. Understanding the implications of continuity is crucial in evaluating the converse's validity.
KF33
Messages
19
Reaction score
0

Homework Statement


I was given the problem of determining if the Converse of the Intermediate Value Theorem in my book was true. Below is my theorem from the book.

Homework Equations

The Attempt at a Solution


I had looked at the converse and tried to draw some examples, and I am thinking it is false. I am leaning that way, because technically the function may or may not be continuous. I just need to know if I am on the right direction.
 

Attachments

  • card-26189841-back.jpg
    card-26189841-back.jpg
    21 KB · Views: 554
Physics news on Phys.org
KF33 said:

Homework Statement


I was given the problem of determining if the Converse of the Intermediate Value Theorem in my book was true. Below is my theorem from the book.

Homework Equations

The Attempt at a Solution


I had looked at the converse and tried to draw some examples, and I am thinking it is false. I am leaning that way, because technically the function may or may not be continuous. I just need to know if I am on the right direction.

What, exactly, would be the converse of the intermediate-value theorem?
 
Ray Vickson said:
What, exactly, would be the converse of the intermediate-value theorem?
If there is at least one number c in [a,b] such that f(c)=k, then f is continuous on the closed interval [a,b] and k is any number between f(a) and f(b). I got the answer though I think.
 
Question: A clock's minute hand has length 4 and its hour hand has length 3. What is the distance between the tips at the moment when it is increasing most rapidly?(Putnam Exam Question) Answer: Making assumption that both the hands moves at constant angular velocities, the answer is ## \sqrt{7} .## But don't you think this assumption is somewhat doubtful and wrong?

Similar threads

  • · Replies 2 ·
Replies
2
Views
2K
  • · Replies 2 ·
Replies
2
Views
1K
  • · Replies 14 ·
Replies
14
Views
2K
  • · Replies 8 ·
Replies
8
Views
2K
  • · Replies 3 ·
Replies
3
Views
2K
Replies
6
Views
2K
  • · Replies 3 ·
Replies
3
Views
1K
  • · Replies 2 ·
Replies
2
Views
3K
  • · Replies 2 ·
Replies
2
Views
2K
  • · Replies 13 ·
Replies
13
Views
2K